Difference between revisions of "2019 AMC 12B Problems/Problem 16"

(Solution)
(Solution)
Line 3: Line 3:
 
==Solution==
 
==Solution==
 
We solve for the probability by doing <math>\frac{1-(Probability of Equality)}{2}</math>.
 
We solve for the probability by doing <math>\frac{1-(Probability of Equality)}{2}</math>.
 +
 +
We see that the probability of equality is the summation of all the probabilities that the balls land in the same container. Thus we have the probability of equality being equal to <math>(\frac{1}{2})(\frac{1}{2})+(\frac{1}{4})(\frac{1}{4})+(\frac{1}{16})(\frac{1}{16})...</math>
 +
 +
The summation of this expression is equal to <cmath>\sum_{n=0}^{\infty} (1)(\frac{1}{4})^{n}-1</cmath>. Using the geometric sum formula, we obtain the summation of this expression to be <math>\frac{1}{\frac{3}{4}}-1</math> or <math>\frac{1}{3}</math>.
  
 
==See Also==
 
==See Also==
 
{{AMC12 box|year=2019|ab=B|num-b=15|num-a=17}}
 
{{AMC12 box|year=2019|ab=B|num-b=15|num-a=17}}

Revision as of 14:12, 14 February 2019

Problem

Solution

We solve for the probability by doing $\frac{1-(Probability of Equality)}{2}$.

We see that the probability of equality is the summation of all the probabilities that the balls land in the same container. Thus we have the probability of equality being equal to $(\frac{1}{2})(\frac{1}{2})+(\frac{1}{4})(\frac{1}{4})+(\frac{1}{16})(\frac{1}{16})...$

The summation of this expression is equal to \[\sum_{n=0}^{\infty} (1)(\frac{1}{4})^{n}-1\]. Using the geometric sum formula, we obtain the summation of this expression to be $\frac{1}{\frac{3}{4}}-1$ or $\frac{1}{3}$.

See Also

2019 AMC 12B (ProblemsAnswer KeyResources)
Preceded by
Problem 15
Followed by
Problem 17
1 2 3 4 5 6 7 8 9 10 11 12 13 14 15 16 17 18 19 20 21 22 23 24 25
All AMC 12 Problems and Solutions